This week, we learn about Vieta Root Jumping, a descent method which uses Vieta's formula to find additional solutions.
You should first read up on Vieta’s Formula.
How would you use Vieta Root Jumping to solve the following?
[IMO 2007/5] Let and be positive integers. Show that if , then .
Share a problem which uses the Vieta root jumping technique.
Easy Math Editor
This discussion board is a place to discuss our Daily Challenges and the math and science related to those challenges. Explanations are more than just a solution — they should explain the steps and thinking strategies that you used to obtain the solution. Comments should further the discussion of math and science.
When posting on Brilliant:
*italics*
or_italics_
**bold**
or__bold__
paragraph 1
paragraph 2
[example link](https://brilliant.org)
> This is a quote
\(
...\)
or\[
...\]
to ensure proper formatting.2 \times 3
2^{34}
a_{i-1}
\frac{2}{3}
\sqrt{2}
\sum_{i=1}^3
\sin \theta
\boxed{123}
Comments
Essentially, Mysterious 100 Degree Monic Polynomial shows we can even Vieta Jump for polynomials.
Another Exercise: Assume that m and n are odd integers such that
m2−n2+1∣n2−1.
Prove that m2−n2+1 is a perfect square.
Yet Another Exercise: Determine all pairs of positive integers (a,b) such that
2ab2−b3+1a2
is a positive integer.
Log in to reply
Indeed. This post only begins to scratch the surface of ideas like this, which in itself is a special case of Fermat's method of Infinite Descent.
The Markov Spectrum, in which we approximate real numbers with rationals, is a non-trivial application of VTJ.
VTJ can also be applied to equations in 3 variables, like x2+y2+z2=3xyz+2.
This one is one of my favorites!
Start out with noting that because gcd(b,4ab−1)=1, we have: 4ab−1∣(4a2−1)2 ⟺4ab−1∣b2(4a2−1)2 ⟹4ab−1∣16a4b2−8a2b2+b2 ⟹4ab−1∣(16a2b2)(a2)−(4ab)(2ab)+b2 ⟹4ab−1∣(1)(a2)−(1)(2ab)+b2 ⟹4ab−1∣(a−b)2 The last step follows from 16a2b2≡(4ab)2≡1(mod4ab−1) and 4ab≡1(mod4ab−1).
Let (a,b)=(a1,b1) be a solution to 4ab−1∣(a−b)2 with a1>b1 contradicting a=b where a1 and b1 are both positive integers. Assume a1+b1 has the smallest sum among all pairs (a,b) with a>b , and I will prove this is absurd. To do so, I prove that there exists another solution (a,b)=(a2,b1) with a smaller sum. Set k=4ab1−1(a−b1)2 be an equation in a. Expanding this we arrive at 4ab1k−k=a2−2ab1+b12 ⟹a2−a(2b1+4b1k)+b12+k=0 This equation has roots a=a1,a2 so we can now use Vieta’s on the equation to attempt to prove that a1>a2. First, we must prove a2 is a positive integer. Notice that from a1+a2=2b1+4b1k via Vieta’s hence a2 is an integer. Assume that a2 is negative or zero. If a2 is zero or negative, then we would have a12−a1(2b1+4b1k)+b12+k=0≥b2+k absurd. Therefore, a2 is a positive integer and (a2,b1) is another pair that contradicts a=b. Now, a1a2=b12+k from Vieta's. Therefore, a2=a1b2+k. We desire to show that a2<a1. a2<a1 ⟺a1b12+k<a1 ⟺b12+4a1b1−1(a1−b1)2<a12 ⟺4a1b1−1(a1−b1)2<(a1−b1)(a1+b1) [ \iff \frac{(a1-b1)}{4a1b1-1} < a1+b1 ] Notice that we can cancel a1−b1 from both sides because we assumed that a1>b1. The last inequality is true because 4a1b1−1>1 henceforth we have arrived at the contradiction that a1+b1>a2+b1. Henceforth, it is impossible to have a>b (our original assumption) and by similar logic it is impossible to have b>a forcing a=b □
We want to force out a nice factorisation from 4ab−1∣(4a2−1)2. A few hit and trials lead me to consider:
b2(4a2−1)2−(4ab−1)(4a3b−2ab+a2)=(a−b)2.
So now, assume that there indeed exists distinct (a,b) that satisfy 4ab−1∣(a−b)2.
Lemma: Suppose (a,b) is a distinct positive integer solution to 4ab−1(a−b)2=k where WLOG a>b. Then (b,ab2+k) is also a solution.
Proof: Remark first that (a−b)2=(4ab−1)k⇔a2−(2b+4kb)a+b2+k=0. By Vieta's Formula for Roots, ab2+k=2b+4kb∈Z+ whence (b,ab2+k) is also a solution. □
However, we easily see that ab2+k=b(2+4k)>b since by assumption k is positive. This means that given a solution (a,b) where b is taken to be minimum, we can always vieta jump to a smaller solution, a clear contradiction. ■
Problem: Show that if x,y,z are positive integers, then (xy+1)(yz+1)(zx+1) is a perfect square if and only if xy+1,yz+1,zx+1 are all perfect squares.
Log in to reply
So, here's my proposed solution.
Solution: It is trivial how to prove the 'if' part. Let us proceed to prove the 'only if' part. Suppose x,y,z are such integers with xy+1,yz+1,zx+1 not all squares. Let us also assume that x,y,z is the smallest counterexample, that is, x+y+z is minimum.
WLOG, xy+1 is not a perfect square. As Dinesh Chavan wrote below, let s be the smallest positive root of s2+x2+y2+z2−2(xy+yz+zs+sx+zx+sy)−4xyzs−4=0. This can be, as he said, written equivalently as:
(x+y−z−s)2=4(xy+1)(zs+1),(x+z−y−s)2=4(xz+1)(ys+1),(x+s−y−z)2=4(xs+1)(yz+1).
But then, our quadratic formula has root s=x+y+z+2xyz±2(xy+1)(yz+1)(zx+1) which was given to be an integer, so we have that RHS is a square. This implies that (xs+1)(ys+1)(zs+1) is a square.
Since, xs+1≥0,ys+1≥0,zs+1≥0, we have that by verifying that x=y=z=1 is not a solution, s≥−max(x,y,z)1>−1.
If s=0, by crunching out the algebra one gets that (x+y−z)2=4(xy+1) which implies that xy+1 is a square, a clear contradiction.
So we have s>0 and by the assumed minimality of x+y+z we can safely conclude that s≥z. Let the 2 roots be s,s1. By Vieta, we have:
ss1=x2+y2+z2−2xy−2yz−2zx−4<z2−x(2z−x)−y(2z−y)<z2
a clear contradiction. ■
Now I hope that Ram Sharma sees the connection to Vieta Jumping.
Log in to reply
@Anqi Li Can you add this to the Vieta Root Jumping Wiki? Thanks!
Hi, this is a nice question. First We state a lemma
Lemma: If (x,y,z) is a P set, then so is (x,y,z,s) for s=x+y+z+2xyz±2(xy+1)(yz+1)(xz+1) as long as s is positive.
Proof: Note that the value of s mentioned is the root of the equation x2+y2+z2+s2−2(xy+yz+zx+xs+ys+zs)−4xyzs−4=0 and now the above quadratic can be written as follows;
(x+y−z−s)2=4(xy+1)(zs+1) Also it can be written as; (x+z−y−s)2=4(xz+1)(ys+1) which can again be written as (x+s−y−z)2=4(xs+1)(yz+1) Since, xy+1 is an integer which is the quotient of two perfect squares, it is also a square. Similarly we can show for yz+1 and zx+1. Which shows that they all must be perfect squares.
Log in to reply
I don't know what the above question has to do with Vieta Root Jumping. However very well explained.+1 for the solution.Is there any other method to prove the result.
Can you define your terms? What is a P set? Does it use 3 variables or 4 variables?
How is xy+1 a quotient of 2 perfect squares? Is zs+1 a perfect square? If so, why?